Question and Answers Forum

All Questions      Topic List

Coordinate Geometry Questions

Previous in All Question      Next in All Question      

Previous in Coordinate Geometry      Next in Coordinate Geometry      

Question Number 150056 by ajfour last updated on 09/Aug/21

Commented by ajfour last updated on 09/Aug/21

If the blue region is a square  of maximum area, find a  and side s of the square.

$${If}\:{the}\:{blue}\:{region}\:{is}\:{a}\:{square} \\ $$$${of}\:{maximum}\:{area},\:{find}\:{a} \\ $$$${and}\:{side}\:{s}\:{of}\:{the}\:{square}.\:\:\: \\ $$

Answered by mr W last updated on 09/Aug/21

y=(x−a)^2 =x  x^2 −(2a+1)x+a^2 =0  s=x=((2a+1−(√(4a+1)))/2)  (ds/da)=0  a−(2/( (√(4a+1))))=0  a^2 =(4/(4a+1))  (1/a^3 )−(1/(4a))−1=0  (1/a)=((((√(1293))/(72))+(1/2)))^(1/3) −((((√(1297))/(72))−(1/2)))^(1/3)   ⇒a≈0.9232

$${y}=\left({x}−{a}\right)^{\mathrm{2}} ={x} \\ $$$${x}^{\mathrm{2}} −\left(\mathrm{2}{a}+\mathrm{1}\right){x}+{a}^{\mathrm{2}} =\mathrm{0} \\ $$$${s}={x}=\frac{\mathrm{2}{a}+\mathrm{1}−\sqrt{\mathrm{4}{a}+\mathrm{1}}}{\mathrm{2}} \\ $$$$\frac{{ds}}{{da}}=\mathrm{0} \\ $$$${a}−\frac{\mathrm{2}}{\:\sqrt{\mathrm{4}{a}+\mathrm{1}}}=\mathrm{0} \\ $$$${a}^{\mathrm{2}} =\frac{\mathrm{4}}{\mathrm{4}{a}+\mathrm{1}} \\ $$$$\frac{\mathrm{1}}{{a}^{\mathrm{3}} }−\frac{\mathrm{1}}{\mathrm{4}{a}}−\mathrm{1}=\mathrm{0} \\ $$$$\frac{\mathrm{1}}{{a}}=\sqrt[{\mathrm{3}}]{\frac{\sqrt{\mathrm{1293}}}{\mathrm{72}}+\frac{\mathrm{1}}{\mathrm{2}}}−\sqrt[{\mathrm{3}}]{\frac{\sqrt{\mathrm{1297}}}{\mathrm{72}}−\frac{\mathrm{1}}{\mathrm{2}}} \\ $$$$\Rightarrow{a}\approx\mathrm{0}.\mathrm{9232} \\ $$

Commented by ajfour last updated on 09/Aug/21

thank u sir, easy 4 u.

$${thank}\:{u}\:{sir},\:{easy}\:\mathrm{4}\:{u}. \\ $$

Commented by ajfour last updated on 09/Aug/21

no it is not so, sir.  for a given a, the area might  be max. for a rectangle;  And for this maximum area  rectangle, to be a square as well  , there has to be some specific   a value, and hence s value.

$${no}\:{it}\:{is}\:{not}\:{so},\:{sir}. \\ $$$${for}\:{a}\:{given}\:{a},\:{the}\:{area}\:{might} \\ $$$${be}\:{max}.\:{for}\:{a}\:{rectangle}; \\ $$$${And}\:{for}\:{this}\:{maximum}\:{area} \\ $$$${rectangle},\:{to}\:{be}\:{a}\:{square}\:{as}\:{well} \\ $$$$,\:{there}\:{has}\:{to}\:{be}\:{some}\:{specific} \\ $$$$\:{a}\:{value},\:{and}\:{hence}\:{s}\:{value}. \\ $$

Commented by mr W last updated on 09/Aug/21

but in fact there is no absolute  maximum or minimum. since  s→∞ when a→∞  s→0 when a→0

$${but}\:{in}\:{fact}\:{there}\:{is}\:{no}\:{absolute} \\ $$$${maximum}\:{or}\:{minimum}.\:{since} \\ $$$${s}\rightarrow\infty\:{when}\:{a}\rightarrow\infty \\ $$$${s}\rightarrow\mathrm{0}\:{when}\:{a}\rightarrow\mathrm{0} \\ $$

Commented by mr W last updated on 09/Aug/21

the upper right corner of the square  on the parabola is the intersection  point of y=(x−a)^2  and y=x.  we see the intersection point always  exists for any value of a from 0 till ∞.

$${the}\:{upper}\:{right}\:{corner}\:{of}\:{the}\:{square} \\ $$$${on}\:{the}\:{parabola}\:{is}\:{the}\:{intersection} \\ $$$${point}\:{of}\:{y}=\left({x}−{a}\right)^{\mathrm{2}} \:{and}\:{y}={x}. \\ $$$${we}\:{see}\:{the}\:{intersection}\:{point}\:{always} \\ $$$${exists}\:{for}\:{any}\:{value}\:{of}\:{a}\:{from}\:\mathrm{0}\:{till}\:\infty. \\ $$

Commented by mr W last updated on 09/Aug/21

if you mean the case that the square  happens to be the maximum rectangle,  then there is such a value for a.

$${if}\:{you}\:{mean}\:{the}\:{case}\:{that}\:{the}\:{square} \\ $$$${happens}\:{to}\:{be}\:{the}\:{maximum}\:{rectangle}, \\ $$$${then}\:{there}\:{is}\:{such}\:{a}\:{value}\:{for}\:{a}. \\ $$

Answered by mr W last updated on 09/Aug/21

say the upper right corner of the  rectangle is (p,(p−a)^2 ).  area of rectangle is  A=p(p−a)^2   (dA/dp)=(p−a)^2 +2p(p−a)=0  p=(a/3)  such that the maximum rectangle  is a square,  p=(p−a)^2   (a/3)=((a/3)−a)^2   ⇒a=(3/4)

$${say}\:{the}\:{upper}\:{right}\:{corner}\:{of}\:{the} \\ $$$${rectangle}\:{is}\:\left({p},\left({p}−{a}\right)^{\mathrm{2}} \right). \\ $$$${area}\:{of}\:{rectangle}\:{is} \\ $$$${A}={p}\left({p}−{a}\right)^{\mathrm{2}} \\ $$$$\frac{{dA}}{{dp}}=\left({p}−{a}\right)^{\mathrm{2}} +\mathrm{2}{p}\left({p}−{a}\right)=\mathrm{0} \\ $$$${p}=\frac{{a}}{\mathrm{3}} \\ $$$${such}\:{that}\:{the}\:{maximum}\:{rectangle} \\ $$$${is}\:{a}\:{square}, \\ $$$${p}=\left({p}−{a}\right)^{\mathrm{2}} \\ $$$$\frac{{a}}{\mathrm{3}}=\left(\frac{{a}}{\mathrm{3}}−{a}\right)^{\mathrm{2}} \\ $$$$\Rightarrow{a}=\frac{\mathrm{3}}{\mathrm{4}} \\ $$

Commented by ajfour last updated on 09/Aug/21

Yes, this is answer to my  question, sir. thanks for  all efforts. really good.

$${Yes},\:{this}\:{is}\:{answer}\:{to}\:{my} \\ $$$${question},\:{sir}.\:{thanks}\:{for} \\ $$$${all}\:{efforts}.\:{really}\:{good}. \\ $$

Terms of Service

Privacy Policy

Contact: info@tinkutara.com